LSAT and Law School Admissions Forum

Get expert LSAT preparation and law school admissions advice from PowerScore Test Preparation.

 Administrator
PowerScore Staff
  • PowerScore Staff
  • Posts: 8919
  • Joined: Feb 02, 2011
|
#47234
Please post your questions below!
 ahhe223
  • Posts: 8
  • Joined: Jun 14, 2017
|
#48160
Premise + Answer = Conclusion
If a new strain has infected 1+ people + (E) no delcline in number of infections due to last year's strains = More local cases of flu infection this year than last year
 Adam Tyson
PowerScore Staff
  • PowerScore Staff
  • Posts: 5153
  • Joined: Apr 14, 2011
|
#48183
That is a perfect example of the correct application of the Justify Formula, ahhe223, and a correct answer! Well done! Did you have any questions about that?
 kliu49
  • Posts: 11
  • Joined: Jan 13, 2019
|
#61824
Hello,

I am confused as to why C is not the correct answer. Using the mechanistic approach, I expected the correct answer to involve the new strain from this year (last year was already mentioned in the premise and conclusion). C makes sense, since if the new strain cannot be resolved, then it would add to the number of this year's cases.

Please help explain, thanks a lot!
Kat
 Robert Carroll
PowerScore Staff
  • PowerScore Staff
  • Posts: 1783
  • Joined: Dec 06, 2013
|
#61865
Kat,

Nothing in answer choice (C) explains what "approaches used to deal with strains" means. If the approaches prevent infections, then having no new approach may mean that more infections occur. But the approaches may be approaches for treatment of infections. If that's the case, then not having a new approach doesn't mean that more infections will occur, but that the cases of infections that do occur won't be handled very well. Note that the conclusion is about how many cases will exist. For answer choice (C) to work, it would at least have to show that the approaches relate to whether the cases come up in the first place, rather than possibly how already-existing cases are treated.

Even worse for this answer, the new strain has already caused cases, so whether new approaches would have prevented those cases is irrelevant anyway. The stimulus already claims that the new strain has made cases of infection occur.

Robert Carroll
 g_lawyered
  • Posts: 211
  • Joined: Sep 14, 2020
|
#95066
Hi P.S.
I had difficult time with this question as I had 2 incorrect answer choices as contenders and eliminated the correct answer. I predicted that the assumption would mention that there's a significant difference between the 2 strains (last year and this year). For this reason, I had answer choice C and D because they both mention differences between the strains that strengthen the conclusion (that there will be more cases of flu this year as opposed to last year). I read the previous explanation of using Justify Formula to prove E correct. I'm also trying to improve my skills in answering this question with the Justify formula.

Using the Justify formula to test C reads:
In addition to the strains of flu that were present in this area last year, a new strain has infected some people this year. (Premise) + (C): The new strain of flu cannot be addressed with the approaches used to deal with the strains of flu that were present last year= (Conclusion): There will be more local cases of flu infection this year than there were last year.
This answer confirms that because there are different approaches to the different strains of flu, that there will be more strains this year than last year. Why or what makes answer C incorrect?

Using the Justify formula to test D reads:
In addition to the strains of flu that were present in this area last year, a new strain has infected some people this year (Premise) + (D): The new strain of flu is expected to be more dangerous than the strains of flu that were present last year= (Conclusion): There will be more local cases of flu infection this year than there were last year.
This also confirms the conclusion. Why or what makes answer D incorrect?

Please help me understand how to correctly use the Justify formula to test out my answer choice contenders. :-?
Thanks in advance
 Robert Carroll
PowerScore Staff
  • PowerScore Staff
  • Posts: 1783
  • Joined: Dec 06, 2013
|
#95202
g_lawyered,

To answer choice (C):

We're not trying to prove there will be more strains, but more cases. Why will the fact that new strains can't be addressed with the same approaches mean more people will get infected in the first place?

Same thing with answer choice (D). Why will more people get infected because the strain is more dangerous?

Robert Carroll
 g_lawyered
  • Posts: 211
  • Joined: Sep 14, 2020
|
#95236
Hi Robert,
Thanks for your reply. I think I see where you're going with your explanation but I need a little more to fully understand. PS Analytics has marked this question as CE- Casual Reasoning. Is the casual reasoning in this argument: New strain (more strain) is the cause and the effect is more local flu cases (from the conclusion).

In my question about answer C:
Using the Justify formula to test C reads:
In addition to the strains of flu that were present in this area last year, a new strain has infected some people this year. (Premise) + (C): The new strain of flu cannot be addressed with the approaches used to deal with the strains of flu that were present last year= (Conclusion): There will be more local cases of flu infection this year than there were last year.
This answer confirms that because there are different approaches to the different strains of flu, that there will be more strains this year than last year.
And your reply
Why will the fact that new strains can't be addressed with the same approaches mean more people will get infected in the first place?
This is where I believe the casual reasoning is supported/proven. Isn't the fact that there will be more strains leads to more flu cases (as the conclusion states)? Answer C states: "The new strain of flu cannot be addressed with the approaches used to deal with the strains of flu that were present last year", so because the approach to the new strains are different doesn't that mean that the new strain is harder to treat/cure, resulting in more flu cases. Is my understanding of the impact of answer C to the conclusion wrong? Or is my analysis on using the Justify Formula incorrect? Am I making an incorrect assumption? Please help me understand why C is incorrect. :-?

In answer D, your explanation
Why will more people get infected because the strain is more dangerous?
I think this confirms the conclusion because there are more people having flu cases because the strain is more dangerous than the previous one. Because a more dangerous strain could mean that there isn't an effective treatment that will 100% get rid of the flu (because the flu cases aren't decreasing/getting rid of. The flu cases are increasing). Maybe this is another incorrect assumption? Is this what makes D incorrect? I think C better justifies the conclusion by confirming the causality better than answer D.
I appreciate your help. Thanks in advance!
 Rachael Wilkenfeld
PowerScore Staff
  • PowerScore Staff
  • Posts: 1358
  • Joined: Dec 15, 2011
|
#95298
Hi g-lawyered,

The issue with both of your answer choices is that they don't get us to the numbers issue in the conclusion. Our conclusion is focused on the NUMBER of local flu infections. We need an answer choice to prove the conclusion that that number will be increasing this year.

Answer choice (C) isn't enough to get us there, because it doesn't address how we treat the old strains of flu. Even if it's true that we can't use the same approaches on the new strain, if we've completely eradicated the old strain, our conclusion wouldn't follow. Similarly, if we have found a completely effective method of blocking the new strain, even if that method is different than the methods we used to control the flu last year, the conclusion wouldn't follow. In sum, this answer choice does not require that the number of flu infections increases because it allows for other ways to control flu numbers.

Answer choice (D) has nothing to do with numbers of cases at all. Dangerous does not equal case numbers. A great real-world example is the disease MERS. MERS is a respiratory disease with a fatality rate of about 35% last I checked. It's a very dangerous illness. However, it's terrible at spreading. Recent studies have shown an infected person likely to not spread it to anyone else. For a comparison, a person infected with the measles will typically spread it to about 16 other people. There's no link made in the argument or in the answer choice between case numbers and dangerous level of the illness. We can't jump from more dangerous to more cases.

Answer choice (E) does deal with numbers. It says that our numbers of the flus from last year will not decrease. That means that the new strain will be an increase in number on any flu cases we had last year. That is enough to prove there will be more flu cases. Flu cases last year + new flu strain cases=more overall cases.

Hope that helps!
 g_lawyered
  • Posts: 211
  • Joined: Sep 14, 2020
|
#96132
Hi Rachael!
I completely missed the argument to be about numbers. Now I understand why both C and D were incorrect answer choices. Thanks for clarifying that!

Get the most out of your LSAT Prep Plus subscription.

Analyze and track your performance with our Testing and Analytics Package.